K
Khách

Hãy nhập câu hỏi của bạn vào đây, nếu là tài khoản VIP, bạn sẽ được ưu tiên trả lời.

2 tháng 9 2020

1 bài BĐT rất hay !!!!!!

BẠN PHÁ TOANG RA HẾT NHÁ SAU ĐÓ THÌ ĐƯỢC CÁI NÀY :33333

\(S=15\left(a^3+b^3+c^3\right)+6\left(a^2b+ab^2+b^2c+bc^2+a^2c+ac^2\right)-72abc\)

\(S=9\left(a^3+b^3+c^3\right)+6\left(a^3+b^3+c^3+a^2b+ab^2+b^2c+bc^2+c^2a+ca^2\right)-72abc\)

\(S=9\left(a^3+b^3+c^3\right)+6\left(a+b+c\right)\left(a^2+b^2+c^2\right)-72abc\)

TA ÁP DỤNG BĐT CAUCHY 3 SỐ SẼ ĐƯỢC:

\(\hept{\begin{cases}a+b+c\ge3\sqrt[3]{abc}\\a^2+b^2+c^2\ge3\sqrt[3]{a^2b^2c^2}\end{cases}}\)

=>    \(\left(a+b+c\right)\left(a^2+b^2+c^2\right)\ge9abc\)

=>    \(72abc\le8\left(a+b+c\right)\left(a^2+b^2+c^2\right)\)

=>   \(-72abc\ge-8\left(a+b+c\right)\left(a^2+b^2+c^2\right)\)

=>   \(S\ge9\left(a^3+b^3+c^3\right)+6\left(a+b+c\right)\left(a^2+b^2+c^2\right)-8\left(a+b+c\right)\left(a^2+b^2+c^2\right)\)

=>   \(S\ge9\left(a^3+b^3+c^3\right)-2\left(a+b+c\right)\left(a^2+b^2+c^2\right)\)

=>   \(S\ge9\left(a^3+b^3+c^3\right)-\frac{2}{9}\left(a+b+c\right)\)

TA LẠI TIẾP TỤC ÁP DỤNG BĐT SAU:   \(\left(a+b+c\right)^2\le3\left(a^2+b^2+c^2\right)\Rightarrow\left(a+b+c\right)^2\le\frac{1}{3}\Rightarrow a+b+c\le\sqrt{\frac{1}{3}}\)

=>   \(S\ge9\left(a^3+b^3+c^3\right)-\frac{2}{9}.\sqrt{\frac{1}{3}}\)

TA LẦN LƯỢT ÁP DỤNG BĐT CAUCHY 3 SỐ SẼ ĐƯỢC:

\(a^3+a^3+\left(\sqrt{\frac{1}{27}}\right)^3\ge3a^2.\sqrt{\frac{1}{27}}\)

ÁP DỤNG TƯƠNG TỰ VỚI 2 BIẾN b; c ta sẽ được 1 BĐT như sau: 

=>   \(2\left(a^3+b^3+c^3\right)+3\left(\sqrt{\frac{1}{27}}\right)^3\ge\frac{3}{\sqrt{27}}\left(a^2+b^2+c^2\right)=\frac{3}{\sqrt{27}}.\left(\frac{1}{9}\right)=\frac{\sqrt{3}}{27}\)

=>   \(a^3+b^3+c^3\ge\frac{\left(\frac{\sqrt{3}}{27}-3\left(\sqrt{\frac{1}{27}}\right)^3\right)}{2}\)

=>   \(S\ge\frac{9\left(\frac{\sqrt{3}}{27}-3\left(\sqrt{\frac{1}{27}}\right)^3\right)}{2}-\frac{2}{9}.\sqrt{\frac{1}{3}}\)

=>   \(S\ge\frac{1}{\sqrt{3}}\)

VẬY TA CÓ ĐPCM.

DẤU "=" XẢY RA <=>   \(a=b=c=\sqrt{\frac{1}{27}}\)

18 tháng 6 2018

\(P=\frac{a}{2b+2c-a}+\frac{b}{2c+2a-b}+\frac{c}{2a+2b-c}=\frac{a^2}{2ab+2ac-a^2}+\frac{b^2}{2bc+2ab-b^2}+\frac{c^2}{2ac+2bc-c^2}\)

vì a,b,c là 3 cạnh của 1 tam giác áp dụng bđt tam giác có:

\(\hept{\begin{cases}b+c>a\Rightarrow2b+2c>a\Rightarrow2ab+2ac>a^2\Rightarrow2ab+2ac-a^2>0\\c+a>b\Rightarrow2c+2a>b\Rightarrow2bc+2ab>b^2\Rightarrow2bc+2ab-b^2>0\\a+b>c\Rightarrow2a+2b>c\Rightarrow2ac+2bc>c^2\Rightarrow2ac+2bc-c^2>0\end{cases}}\)

\(\Rightarrow\frac{a^2}{2ab+2ac-a^2}+\frac{b^2}{2bc+2ab-b^2}+\frac{c^2}{2ac+2bc-c^2}>0\)áp dụng bđt cauchy schawazt dạng enge ta có:

\(\frac{a^2}{2ab+2ac-a^2}+\frac{b^2}{2bc+2ab-b^2}+\frac{c^2}{2ac+2bc-c^2}>=\)

\(\frac{\left(a+b+c\right)^2}{2ab+2ac-a^2+2bc+2ab-b^2+2ac+2bc-c^2}=\frac{\left(a+b+c\right)^2}{4ab+4ac+4bc-\left(a^2+b^2+c^2\right)}\left(1\right)\)

vì \(a^2+b^2+c^2>=ab+ac+bc\Rightarrow4ab+4ac+4bc-\left(a^2+b^2+c^2\right)< =\)

\(4ab+4ac+4bc-\left(ab+ac+bc\right)\)mà \(\left(a+b+c\right)^2>0\)

\(\Rightarrow\frac{\left(a+b+c\right)^2}{4ab+4ac+4bc-\left(a^2+b^2+c^2\right)}>=\frac{\left(a+b+c\right)^2}{4ab+4ac+4bc-\left(ab+ac+bc\right)}\)(2)

\(=\frac{\left(a+b+c\right)^2}{4ab+4ac+4bc-ab-ac-bc}=\frac{\left(a+b+c\right)^2}{3ab+3ac+3bc}=\frac{a^2+b^2+c^2+2ab+2ac+2bc}{3ab+3ac+3bc}\)

\(>=\frac{ab+ac+bc+2ab+2ac+2bc}{3ab+3ac+3bc}=\frac{3ab+3ac+3bc}{3ab+3ac+3bc}=1\)(3)

từ (1)(2)(3)\(\Rightarrow\frac{a^2}{2ab+2ac-a^2}+\frac{b^2}{2bc+2ab-b^2}+\frac{c^2}{2ac+2bc-c^2}>=1\)

\(\Rightarrow P=\frac{a}{2b+2c-a}+\frac{b}{2c+2a-b}+\frac{c}{2a+2b-c}>=1\)

dấu = xảy ra khi a=b=c

vậy min P là 1 khi a=b=c

Đặt \(\hept{\begin{cases}x=2b+2c-a\\y=2c+2a-b\\z=2a+2b-c\end{cases}}\)

Vì a,b,c là độ dài ba cạnh của 1 tam giác nên \(x,y,z>0\)

Khi đó :

\(\Rightarrow\hept{\begin{cases}a=\frac{2y+2z-x}{9}\\b=\frac{2z+2x-y}{9}\\c=\frac{2x+2y-z}{9}\end{cases}}\)

Ta có bất đẳng thức mới theo ẩn x,y,z : 

\(\frac{2y+2z-x}{9x}+\frac{2z+2x-y}{9y}+\frac{2x+2y-z}{9z}\ge1\)

\(\Leftrightarrow\frac{2}{9}\left(\frac{y}{x}+\frac{z}{x}\right)+\frac{2}{9}\left(\frac{z}{y}+\frac{x}{y}\right)+\frac{2}{9}\left(\frac{x}{z}+\frac{y}{z}\right)-\frac{1}{3}\ge1\)

\(\Leftrightarrow\frac{2}{9}\left(\frac{x}{y}+\frac{y}{x}\right)+\frac{2}{9}\left(\frac{y}{z}+\frac{z}{y}\right)+\frac{2}{9}\left(\frac{z}{x}+\frac{x}{z}\right)-\frac{1}{3}\ge1\)

Ta chứng minh bất đẳng thức phụ sau : 

\(\frac{a}{b}+\frac{b}{a}\ge2\forall a,b>0\)

Thật vậy : \(\frac{a}{b}+\frac{b}{a}\ge2\)

\(\Leftrightarrow\frac{a^2}{ab}+\frac{b^2}{ab}\ge2\)

\(\Leftrightarrow\frac{a^2+b^2}{ab}-2\ge0\)

\(\Leftrightarrow\frac{a^2+b^2-2ab}{ab}\ge0\)

\(\Leftrightarrow\frac{\left(a-b\right)^2}{ab}\ge0\)(luôn đúng \(\forall a,b>0\))

Áp dụng , ta được :

\(\frac{2}{9}.2+\frac{2}{9}.2+\frac{2}{9}.2-\frac{1}{3}\ge1\)

\(\Leftrightarrow\frac{12}{9}-\frac{1}{3}\ge1\)

\(\Leftrightarrow\frac{9}{9}\ge1\)(đúng)

Vậy bất đẳng thức được chứng minh 

1 tháng 2 2017

a on à :D 

11 tháng 9 2016
Xét tam giác ABC có: AB = c, BC = a, AC = b.Từ A dựng đường thẳng d // BC. Lấy B' đối xứng với B qua d.Ta nhận thấy: BB' = 2.h . Ta có: B B ′ 2 + B C 2 = B ′ C 2 BB′2+BC2=B′C2 \leq ( B ′ A + A C ) 2 (B′A+AC)2 . Suy ra: 4. h a 2 4.ha2 \leq ( c + b ) 2 − a 2 (c+b)2−a2 (1) Hoàn toàn tương tự: 4. h b 2 4.hb2 \leq ( c + a ) 2 − b 2 (c+a)2−b2 (2) 4. h c 2 4.hc2 \leq ( a + b ) 2 − c 2 (a+b)2−c2 (3) Từ (1)(2)(3) ta có: ( c + b ) 2 − a 2 + ( c + a ) 2 − b 2 + ( a + b ) 2 − c 2 (c+b)2−a2+(c+a)2−b2+(a+b)2−c2 \geq 4. ( h a 2 + h b 2 + h c 2 ) 4.(ha2+hb2+hc2) \Rightarrow ( a + b + c ) 2 (a+b+c)2 \geq 4. ( h a 2 + h b 2 + h c 2 ) 4.(ha2+hb2+hc2) (dpcm)
1. Tìm các số tự nhiên a, b, c khác 0 thỏa mãn:\(\frac{28}{29}< \frac{1}{a}+\frac{1}{b}+\frac{1}{c}< 1.\)2. Chứng minh rằng trọng tâm, trực tâm và tâm đường tròn nội tiếp (giao điểm của 3 đường trung trực) trong một tam giác thẳng hàng.3. chứng minh rằng nếu a,b,c là các số hửu tỉ thì \(\sqrt{a}+\sqrt{b}+\sqrt{c}\)là số hửu tỉ.4.Cho tam giác ABC có \(\widehat{A}=30^0\), BC=2cm. Trên cạnh AC lấy điểm D sao...
Đọc tiếp

1. Tìm các số tự nhiên a, b, c khác 0 thỏa mãn:\(\frac{28}{29}< \frac{1}{a}+\frac{1}{b}+\frac{1}{c}< 1.\)

2. Chứng minh rằng trọng tâm, trực tâm và tâm đường tròn nội tiếp (giao điểm của 3 đường trung trực) trong một tam giác thẳng hàng.

3. chứng minh rằng nếu a,b,c là các số hửu tỉ thì \(\sqrt{a}+\sqrt{b}+\sqrt{c}\)là số hửu tỉ.

4.Cho tam giác ABC có \(\widehat{A}=30^0\), BC=2cm. Trên cạnh AC lấy điểm D sao cho \(\widehat{CBD}=60^0\). Tính độ dài AD.

5. Tìm các số a,b sao cho 2007ab là bình phương của số tự nhiên.

6. Cho tam giác ABC vuông tại A, đường cao AH. Gọi M,N lần lượt là trung điểm của AH và BH. Chứng minh rằng \(CM\perp AN\)

7. Chứng minh rằng: \(\frac{1}{\sqrt{1}}+\frac{1}{\sqrt{2}}+\frac{1}{\sqrt{3}}+....+\frac{1}{\sqrt{100}}>10\)

8. Cho tam giác ABC, H là trực tâm, O là tâm đường tròn đi qua ba đỉnh của tam giác. Chứng minh rằng khoảng cách từ O đến một cạnh của tam giác bằng một nửa khoảng cách từ H đến đỉnh đối diện.

9. Tìm x,y,z biết: \(\frac{y+z+1}{x}=\frac{x+z+2}{y}=\frac{x+y-3}{z}=\frac{1}{x+y+z}\)

10. Độ dài ba cạnh của 1 tam giác tỉ lệ với 2;3;4. Hỏi ba chiều cao tương ứng của tam giác đó tỉ lệ với ba số nào?

2
11 tháng 4 2018

Bài 7 : 

( bạn đạt A = (...) cái biểu thức đấy nhé, tự đặt ) 

Ta có : 

\(\frac{1}{\sqrt{1}}=\frac{1}{1}>\frac{1}{10}=\frac{1}{\sqrt{100}}\)

\(\frac{1}{\sqrt{2}}>\frac{1}{\sqrt{100}}\)

\(\frac{1}{\sqrt{3}}>\frac{1}{\sqrt{100}}\)

\(............\)

\(\frac{1}{\sqrt{100}}=\frac{1}{\sqrt{100}}\)

\(\Rightarrow\)\(A=\frac{1}{\sqrt{1}}+\frac{1}{\sqrt{2}}+\frac{1}{\sqrt{3}}+...+\frac{1}{\sqrt{100}}>\frac{1}{\sqrt{100}}+\frac{1}{\sqrt{100}}+\frac{1}{\sqrt{100}}+...+\frac{1}{\sqrt{100}}\)

\(A>\frac{100}{\sqrt{100}}=\frac{100}{10}=10\)

\(\Rightarrow\)\(A>10\)

Vậy \(A>10\)

Chúc bạn học tốt ~ 

11 tháng 4 2018

Bạn làm được mình bài 7 thôi à, mình thấy bạn giỏi lắm mà. Mình có tới mấy chục bài cần giải cơ. Dạo này mình hỏi nhiều vì sắp đi thi.

5 tháng 7 2021

Ta có:

A = \(\frac{a}{2b+3c}+\frac{b}{2c+3a}+\frac{c}{3b+2a}=\frac{a^2}{2ab+3ac}+\frac{b^2}{2bc+3ab}+\frac{c^2}{3bc+2ac}\)

\(\ge\frac{\left(a+b+c\right)^2}{2ab+3ac+2bc+3ab+3bc+2ac}\)(bđt svacxo \(\frac{x_1^2}{y_1}+\frac{x_2^2}{y_2}+\frac{x_3^2}{y_3}\ge\frac{\left(x_1+x_2+x_3\right)^2}{y_1+y_2+y_3}\))

\(\ge\frac{\left(a+b+c\right)^2}{5\left(ab+bc+ac\right)}\ge\frac{\left(a+b+c\right)^2}{\frac{5\left(a+b+c\right)^2}{3}}\) (bđt \(xy+yz+xz\le\frac{\left(x+y+z\right)^2}{3}\)(*)

CM bđt * <=> \(3xy+3yz+3xz\le x^2+y^2+z^2+2xz+2xy+2yz\)

<=> \(\left(x-y\right)^2+\left(x-z\right)^2+\left(y-z\right)^2\ge0\) (luôn đúng)

<=> A \(\ge\frac{3}{5}\) --> ĐPCM